cyruswhittaker
Thanks Received: 107
Forum Guests
 
Posts: 246
Joined: August 11th, 2010
 
 
trophy
Most Thanked
trophy
First Responder
 

Q9 - Museum visitor: The national government

by cyruswhittaker Tue Aug 31, 2010 3:55 pm

I had some lag on this problem and got confused a little bit so it took me longer than I wanted to spend on it.

I got A but just want to verify my thought pattern on this.

The argument states that the 5% increase in minimum wage is going to cause an increase in operating expenses, forcing them to raise admission or decrease service as a result.

The necessary assumption is that the employees aren't already earning well above the 5% increase. (this was my immediate, imprecise assumption that came to mind).

When I select an answer for necessary assumptions, I generally negate/deny the assumption to see if it attacks the conclusion.

Do you have a quick/easy way for denying the assumption with a some statement?

I usually just change "some" to "all" and remove (or add) the "not."

So in this case A would turn into:

ALL of the museum's employees ARE paid significantly more than the minimum wage.
 
giladedelman
Thanks Received: 833
LSAT Geek
 
Posts: 619
Joined: April 04th, 2010
 
This post thanked 3 times.
 
 

Re: Q9 - Museum visitor: The national government

by giladedelman Thu Sep 02, 2010 12:28 pm

Awesome post! Your explanation is spot on. The museum visitor argues that raising the minimum wage will raise museum expenses -- but what if all the employees already make well over the minimum wage? Then it would have no effect!

Strictly speaking, we negate "some" by changing it to "none." That makes sense, right?

"I have some money."

"No you don't, you have none!"

So answer (A), negated, would look like this:

None of the museum's employees are not paid significantly more than the minimum wage.

That leaves us with a double negative: none are not paid. This we would translate to

All of the museum's employees are paid significantly more than the minimum wage.

So, your method of changing "some" to "all" and then adding or subtracting "not," depending on whether it's there, is valid. However, while you're practicing, I recommend trying to really understand these statements and their negations on a deeper, more natural level. That way you won't have to rely on a formula. Does that make sense?
 
goriano
Thanks Received: 12
Atticus Finch
Atticus Finch
 
Posts: 113
Joined: December 03rd, 2011
 
 
 

Re: PT 54, S2, Q9 - Museum visitor: the national government has

by goriano Sun Jan 29, 2012 9:47 pm

giladedelman Wrote:Awesome post! Your explanation is spot on. The museum visitor argues that raising the minimum wage will raise museum expenses -- but what if all the employees already make well over the minimum wage? Then it would have no effect!

Strictly speaking, we negate "some" by changing it to "none." That makes sense, right?

"I have some money."

"No you don't, you have none!"

So answer (A), negated, would look like this:

None of the museum's employees are not paid significantly more than the minimum wage.

That leaves us with a double negative: none are not paid. This we would translate to

All of the museum's employees are paid significantly more than the minimum wage.

So, your method of changing "some" to "all" and then adding or subtracting "not," depending on whether it's there, is valid. However, while you're practicing, I recommend trying to really understand these statements and their negations on a deeper, more natural level. That way you won't have to rely on a formula. Does that make sense?


Why can't we negate just the verb in answer choice (A)? That is, "some of the museum's employees ARE paid significantly more than the minimum wage." While I do know that SOME can mean ALL on the LSAT, I'm wondering whether I am missing anything here. Thanks!
 
giladedelman
Thanks Received: 833
LSAT Geek
 
Posts: 619
Joined: April 04th, 2010
 
 
 

Re: Q9 - Museum visitor: the national government has

by giladedelman Fri Feb 03, 2012 8:16 am

We can't do that because that doesn't negate the original statement. If we know "some X are Y," that leaves open the possibility that some X are not Y -- there's no conflict there.

Imagine I say, "Some of my friends are from New Jersey." Then I ask you to negate that statement. What if you just negate the verb, as you suggest? Then you'd be saying that some of my friends are not from New Jersey. But how is that a problem? I can have friends from New Jersey and friends from California. You haven't negated my statement. The only way to negate it is to say that none of my friends are from New Jersey, because then it's definitely not true that some of them are.

Does that answer your question?
 
timmydoeslsat
Thanks Received: 887
Atticus Finch
Atticus Finch
 
Posts: 1136
Joined: June 20th, 2011
 
This post thanked 5 times.
 
trophy
Most Thanked
trophy
First Responder
 

Re: Q9 - Museum visitor: the national government has

by timmydoeslsat Fri Feb 03, 2012 7:51 pm

I always go by this:

Negation of answer choices:

Conditional statement: A ---> B

Even if A, then ~B.

We are showing that the necessary condition does not have to be necessary, and this negates it. (Technically we would use "even if...then not necessarily B, but this idea gets the point across sufficiently)

Quantifying statement:

Some: Change to none.
Most: Change to half or less.
All: Change to not all.

And the right hand side of those statements can be negated to the left side. The negation of those terms goes both ways.

Convoluted statements in the heat of the test:

It is not the case that [Answer Choice]
User avatar
 
ManhattanPrepLSAT1
Thanks Received: 1909
Atticus Finch
Atticus Finch
 
Posts: 2851
Joined: October 07th, 2009
 
 
 

Re: Q9 - Museum visitor: the national government has

by ManhattanPrepLSAT1 Fri Feb 03, 2012 10:02 pm

nice Timmy!
 
johnsdouglass
Thanks Received: 1
Forum Guests
 
Posts: 5
Joined: July 13th, 2013
 
 
 

Re: Q9 - Museum visitor: The national government

by johnsdouglass Sat Jul 13, 2013 8:35 pm

It is clear why B,C,D and E are wrong, but it is not clear why A is a necessary assumption.

My understanding is the conclusion of the argument is that the increased minimum wage will negatively impact the museum-going public.

One of the premises given in the stimulus for this assertion is that operating expenses will increase.

Taking answer A and applying an increase in minimum wage does increase operating expenses, but this increase in operating expenses is already accepted as fact since it is one of our premises. In this respect, answer A supports one of our premises, but does not seem to be a necessary assumption.

Can someone straighten me out?
 
s.atrmachin3
Thanks Received: 4
Vinny Gambini
Vinny Gambini
 
Posts: 17
Joined: March 05th, 2013
 
 
 

Re: Q9 - Museum visitor: The national government

by s.atrmachin3 Wed Aug 28, 2013 5:52 pm

From my understanding of the question and answer (A), we must assume that some of the employees are currently being paid minimum wage. Otherwise (i.e. all employees already make much more than minimum wage) the mandate has no effect (because it's only increasing minimum wage by 5%). This wipes out the premise that the mandate will increase operating expenses, and our conclusion has no support as a result. Therefore, the argument requires the assumption that is answer choice (A).
 
alena21century
Thanks Received: 1
Vinny Gambini
Vinny Gambini
 
Posts: 9
Joined: January 09th, 2014
 
 
 

Re: Q9 - Museum visitor: The national government

by alena21century Fri Jan 31, 2014 10:40 pm

This one took me a while to figure out. From what I understand, answer choice A is the correct answer since the assumption in the argument is simply that some of the museum employees are eligible for the mandatory 5% raise. Answer A states "some of the museum employees are not paid significantly more than the minimum wage."
Lets say that the minimum wage is $7.00 and it is to be increased to $7.35 (5%). So, if the museum has some employees who make less that $7.35 that would increase their operating expenses even more.
The confusing part for me was the statement in the stimulus that said that the operating expenses would be significantly increased. That would mean that a lot of employees would have to get a raise. Now, in formal logic the word some can mean most or even all. So, "some employees" stated in Answer A could mean most or even all employees. So, if most or all employees would get a raise then the museum would have significant increase in operating expenses.
User avatar
 
Mab6q
Thanks Received: 31
Atticus Finch
Atticus Finch
 
Posts: 290
Joined: June 30th, 2013
 
 
 

Re: Q9 - Museum visitor: The national government

by Mab6q Sun Mar 30, 2014 7:09 pm

johnsdouglass Wrote:It is clear why B,C,D and E are wrong, but it is not clear why A is a necessary assumption.

My understanding is the conclusion of the argument is that the increased minimum wage will negatively impact the museum-going public.

One of the premises given in the stimulus for this assertion is that operating expenses will increase.

Taking answer A and applying an increase in minimum wage does increase operating expenses, but this increase in operating expenses is already accepted as fact since it is one of our premises. In this respect, answer A supports one of our premises, but does not seem to be a necessary assumption.

Can someone straighten me out?


I had the same concern and would appreciate someone chiming in. There's no doubt that A is the right answer, but doesnt it seem somewhat like a premise booster. The stimulus states that the operating costs are going to increase, so this just reaffirms that. That was my hesitation with A.
"Just keep swimming"
 
yingxxgao
Thanks Received: 0
Vinny Gambini
Vinny Gambini
 
Posts: 1
Joined: October 07th, 2013
 
 
 

Re: Q9 - Museum visitor: The national government

by yingxxgao Thu Jul 10, 2014 3:58 pm

Hi Mab6q, in response to your question, I think you are right that that the premise accepts that operating expense will increase with the mandate, and this we will accept as a fact. However, in stating that fact, the museum visitor assumes that the increase in expense will come from increasing minimum wages for workers who hasn't been paid the new minimum wage. This is the necessary assumption that would need to be true in order for the argument to stand.

Answer choice (A) does not restate a part of the premise. It states the assumption behind the "increase in operating expense". Hope this is helpful!
 
smsotolongo
Thanks Received: 1
Jackie Chiles
Jackie Chiles
 
Posts: 33
Joined: September 21st, 2014
 
 
 

Re: Q9 - Museum visitor: The national government

by smsotolongo Thu Jan 01, 2015 6:11 pm

Question on this one. Since the conclusion was "this mandate will adversely affect the museum-going public" I went with an assumption along the lines of there is no projected increase in the number of people attending the museum. I am not understanding why such an assumption is wrong. It's the reason I chose B. Why am I wrong?
User avatar
 
rinagoldfield
Thanks Received: 308
Atticus Finch
Atticus Finch
 
Posts: 390
Joined: December 13th, 2011
 
This post thanked 1 time.
 
 

Re: Q9 - Museum visitor: The national government

by rinagoldfield Sun Jan 04, 2015 5:34 pm

Hi smsotolongo,

Thanks for your post. I can see why (B) is tempting. You are on the right track here- another assumption in this argument is that the museum can’t raise its revenue through any method other than raising fees or decreasing services. A projected increase in attendees could be one way to get new revenue.
A correct answer choice could say something like “there is no significant increase in the number of people purchasing full-price tickets to the museum projected.”

… which is not what (B) says. (B) talks about the past, not the future. What happened for the past five years doesn’t affect the argument; future revenue sources do.

Does that make sense?
 
muriella
Thanks Received: 0
Vinny Gambini
Vinny Gambini
 
Posts: 11
Joined: September 06th, 2011
 
 
 

Re: Q9 - Museum visitor: The national government

by muriella Wed Oct 07, 2015 12:35 pm

Hi - Could a Manhattan geek help clarify (A) again on the issues that the previous two posters - mab6q and johnsdouglass - had?? I had the same issue with it.

Thanks so much .
User avatar
 
ohthatpatrick
Thanks Received: 3805
Atticus Finch
Atticus Finch
 
Posts: 4661
Joined: April 01st, 2011
 
This post thanked 2 times.
 
 

Re: Q9 - Museum visitor: The national government

by ohthatpatrick Mon Oct 12, 2015 12:36 am

When we negate (A), we end up badly weakening the argument by hearing
"ALL the museum's employees are paid significantly more than the minimum wage".

This weakens the argument by undermining the idea "the mandate to increase the minimum wage by 5% will significantly increase the museum's operating expenses".

How could a 5% increase on the minimum wage affect the museum's operating expenses if all the museum's employees currently are paid SIGNIFICANTLY more than minimum wage?

====

Necessary Assumption = "Which of the following, if false, most weakens the argument?"

Here are all the answers, negated. Which one most weakens the argument?

(A) Everyone who works here makes way above minimum wage
(B) Our revenue has varied at least somewhat over the past five years.
(C) Everyone who works here currently makes minimum wage
(D) The number of visitors we have each year has not been a steady increase (maybe sometimes a drastic increase, or no increase, or slight dip, who knows)
(E) All visitors have to pay an admission fee

Hopefully you see why (A) is the clear winner in terms of fighting this argument. "The higher minimum wage is NOT gonna adversely affect the museum, because even the new minimum wage will still be lower than what everyone at the museum already makes ... so the museum won't have to change their pay at all in relation to the new mandate.

=====

What irks all of us about this is that it appears we're weakening the argument by "denying" the truth of a premise.

However, the statement that is being "denied" here is actually an intermediate conclusion. "...the mandate will significantly increase the museum's operating expenses" is an intermediate conclusion supported by the statements that the minimum wage will increase and the museum's revenue does not currently exceed its expenses. And that's where the gap is - from those statements, we can only get to the intermediate conclusion if some of the employees aren't making much more than minimum wage.
 
sh854
Thanks Received: 0
Jackie Chiles
Jackie Chiles
 
Posts: 26
Joined: July 08th, 2013
 
 
 

Re: Q9 - Museum visitor: The national government

by sh854 Wed Oct 14, 2015 11:32 pm

Thank you for the explanation. I thought the opposite of some on the LSAT is none. Thus, I was negating the answer choices with some to none and got the wrong answer. Why is the negation of some ALL?
User avatar
 
ohthatpatrick
Thanks Received: 3805
Atticus Finch
Atticus Finch
 
Posts: 4661
Joined: April 01st, 2011
 
This post thanked 1 time.
 
 

Re: Q9 - Museum visitor: The national government

by ohthatpatrick Sat Oct 17, 2015 4:53 pm

You're correct, the negation of "some" is "none".

But when you do that to (A), you see a double negative:
NONE of the employees are NOT paid more than the minimum wage.

I was just cleaning up the double negative by expressing it as a positive.

If I say, "NONE of my friends HAVEN'T seen 'Star Wars'." then I'm saying that "ALL of my friends HAVE".

No A's are B = All A's are ~B

No [employees] are [not paid more than min wage]
=
All [employees] are NOT [not paid more than min wage]
=
All [employees] are [paid more than min wage]

Does that make sense?
 
seychelles1718
Thanks Received: 0
Atticus Finch
Atticus Finch
 
Posts: 136
Joined: November 01st, 2015
 
 
 

Re: Q9 - Museum visitor: The national government

by seychelles1718 Thu May 04, 2017 6:24 am

I think we can say the question is technically asking us to attack the premise but IMHO the statement that the mandate will significantly increase the operating expense is actually an intermediate conclusion (which is a premise and conclusion at the same time).

Mandated 5 % increase in min wage for all workers --> the mandate will significantly increase the operating expense
 
bsd987
Thanks Received: 0
Forum Guests
 
Posts: 4
Joined: October 21st, 2011
 
 
 

Re: Q9 - Museum visitor: The national government

by bsd987 Thu Jun 01, 2017 10:15 am

I'd forgotten about this question since four of the answers are quite fairly wrong, but one of my students brought it back to my attention yesterday and I'm struggling to accept any of these explanations. It may be 'UNSTATED', but it's not unproven. And even if it were unproven, it's far from necessary.

What have we established?

1. Gov't has mandated a 5 percent increase to minimum wage to all workers.
2. That specific mandate will significantly increase the museum's operating expenses.

Conclusion: Adversely affect museum-going public

There are certainly gaps in this argument. But he has established enough evidence that we can logically infer that the mandate will lead to some (and, likely, a fairly large number of) workers receiving an increase in pay. We accept in flaw-type questions that that is sufficient to prove something, hence why it cannot be considered a flaw that the prompt does not, for instance, provide his source, or related issues. It is why, for instance, on a sufficient assumption question, it is in fact sufficient to accept the truth of the added premise on its face without also adding in 'and that all premises have a solid, unquestionable factual basis'.

But even accepting the argument that we could contradict the premise and that it is an acceptable underlying assumption that the premises are true—something that flies in the face of every other question type—it is not necessary. The necessary assumption would be that at least some people who directly or indirectly work for the museum or companies with which it does business currently do not make significantly more than the minimum wage. The premise leaves a lot of room for maneuvering, and expenditures could just as easily be significantly increased by increased costs they have to pay to contractors (artwork transporters, vendors, advertising agencies, etc) that employee minimum wage employees.

It's clear that B, C, D, and E are wrong, but A is a terrible correct answer.
 
seychelles1718
Thanks Received: 0
Atticus Finch
Atticus Finch
 
Posts: 136
Joined: November 01st, 2015
 
 
 

Re: Q9 - Museum visitor: The national government

by seychelles1718 Tue Jan 16, 2018 3:42 am

ohthatpatrick Wrote:
What irks all of us about this is that it appears we're weakening the argument by "denying" the truth of a premise.

However, the statement that is being "denied" here is actually an intermediate conclusion. "...the mandate will significantly increase the museum's operating expenses" is an intermediate conclusion supported by the statements that the minimum wage will increase and the museum's revenue does not currently exceed its expenses. And that's where the gap is - from those statements, we can only get to the intermediate conclusion if some of the employees aren't making much more than minimum wage.


How do you know that "the mandate will significantly increase the museum's operating expenses" is an intermediate conclusion supported by the statements that the minimum wage will increase and the museum's revenue does not currently exceed its expenses? Those two sentences are connected with "and" within a sentence; doesn't this mean they are separate premises that have no relationship of logic with each other? When you read the stimulus, it's actually very hard to see that those two ideas are in the support-conclusion relationship but rather they completely seem like two separate ideas.